0 Daumen
251 Aufrufe

Beweisen Sie:

\( \sum\limits_{k=0}^{\infty}{\frac{1}{k!}= e} \)  mit e := \( \lim\limits_{n\to\infty} (1 + \frac{1}{n} \))n 

Entwickeln Sie dazu den Term (1+\( \frac{1}{n} \))n mit Hilfe des Binomiallehrsatzes in eine Summe und vergleichen diese mit der n-ten Partialsumme der Reihe \( \sum\limits_{k=0}^{\infty}{\frac{1}{k!}} \)

Avatar von

2 Antworten

+1 Daumen
 
Beste Antwort

\( \sum\limits_{k=0}^{\infty}{\frac{1}{k!}= e} \)  mit e := \( \lim\limits_{n\to\infty} (1 + \frac{1}{n})^n \)

$$(1 + \frac{1}{n})^n = 1^n + \begin{pmatrix} n\\1 \end{pmatrix}*1*\frac{1}{n}+\begin{pmatrix} n\\2 \end{pmatrix}*1*\frac{1}{n^2}+....\begin{pmatrix} n\\n \end{pmatrix}*1*\frac{1}{n^n}$$

Die Summanden sind also alle von der Form

$$ \begin{pmatrix} n\\k \end{pmatrix}*\frac{1}{n^k}=\frac{n*(n-1)**(n-k)}{k!}*\frac{1}{n^k}$$

$$=\frac{n*(n-1)**(n-k)}{n^k}*\frac{1}{k!}=\frac{n}{n}*\frac{n-1}{n}*\frac{n-2}{n}*...\frac{n-k}{n}*\frac{1}{k!}$$

Und für n gegen unendlich gehen die doch alle gegen $$\frac{1}{k!}$$

Also geht (1+1/n)^n gegen die n-te Partialsumme.

Avatar von 288 k 🚀
+1 Daumen

(1+1/n)n=∑(von k=0 bis n) (n!/(k!(n-k)!)·1/nk).

Avatar von 123 k 🚀

Ein anderes Problem?

Stell deine Frage

Willkommen bei der Mathelounge! Stell deine Frage einfach und kostenlos

x
Made by a lovely community